Difference between revisions of "2017 USAJMO Problems/Problem 5"

(Solution)
(Solution)
(One intermediate revision by one other user not shown)
Line 4: Line 4:
  
 
==Solution==
 
==Solution==
[asy] pair A = dir(130); pair B = dir(220); pair C = dir(320); draw(unitcircle, lightblue);
+
<asy>
 +
size(9cm);
 +
pair A = dir(130);  
 +
pair B = dir(220);  
 +
pair C = dir(320);  
 +
draw(unitcircle, lightblue);
  
pair P = dir(-90); pair Q = dir(90); pair D = extension(A, P, B, C); pair O = origin; pair M = extension(B, C, O, P); pair N = 2*M-P;
+
pair P = dir(-90);  
 +
pair Q = dir(90);
 +
pair D = extension(A, P, B, C);  
 +
pair O = origin;  
 +
pair M = extension(B, C, O, P);  
 +
pair N = 2*M-P;
  
draw(A--B--C--cycle, lightblue); draw(A--P--Q, lightblue); draw(A--N--D--O--A, lightblue);
+
draw(A--B--C--cycle, lightblue);  
 +
draw(A--P--Q, lightblue);  
 +
draw(A--N--D--O--A, lightblue);
  
 
draw(A--D--N--O--cycle, red);
 
draw(A--D--N--O--cycle, red);
  
dot("<math>A</math>", A, dir(A)); dot("<math>B</math>", B, dir(B)); dot("<math>C</math>", C, dir(C)); dot("<math>P</math>", P, dir(P)); dot("<math>Q</math>", Q, dir(Q)); dot("<math>D</math>", D, dir(225)); dot("<math>O</math>", O, dir(315)); dot("<math>M</math>", M, dir(315)); dot("<math>N</math>", N, dir(315));
+
dot("$A$", A, dir(A));  
 
+
dot("$B$", B, dir(B));  
/* TSQ Source:
+
dot("$C$", C, dir(C));  
 
+
dot("$P$", P, dir(P));  
A = dir 130 B = dir 220 C = dir 320 unitcircle 0.1 lightcyan / lightblue
+
dot("$Q$", Q, dir(Q));  
 
+
dot("$D$", D, dir(225));  
P = dir -90 Q = dir 90 D = extension A P B C R225 O = origin R315 M = extension B C O P R315 N = 2*M-P R315
+
dot("$O$", O, dir(315));  
 
+
dot("$M$", M, dir(315));  
A--B--C--cycle lightblue A--P--Q lightblue A--N--D--O--A lightblue
+
dot("$N$", N, dir(315));
 
+
</asy>
A--D--N--O--cycle 0.1 yellow / red
 
 
 
[/asy]
 
  
 
Suppose ray <math>OM</math> intersects the circumcircle of <math>BHC</math> at <math>N'</math>, and let the foot of the A-altitude of <math>ABC</math> be <math>E</math>. Note that <math>\angle BHE=90-\angle HBE=90-90+\angle C=\angle C</math>. Likewise, <math>\angle CHE=\angle B</math>. So, <math>\angle BHC=\angle BHE+\angle CHE=\angle B+\angle C</math>.
 
Suppose ray <math>OM</math> intersects the circumcircle of <math>BHC</math> at <math>N'</math>, and let the foot of the A-altitude of <math>ABC</math> be <math>E</math>. Note that <math>\angle BHE=90-\angle HBE=90-90+\angle C=\angle C</math>. Likewise, <math>\angle CHE=\angle B</math>. So, <math>\angle BHC=\angle BHE+\angle CHE=\angle B+\angle C</math>.

Revision as of 22:37, 9 March 2019

Problem

Let $O$ and $H$ be the circumcenter and the orthocenter of an acute triangle $ABC$. Points $M$ and $D$ lie on side $BC$ such that $BM = CM$ and $\angle BAD = \angle CAD$. Ray $MO$ intersects the circumcircle of triangle $BHC$ in point $N$. Prove that $\angle ADO = \angle HAN$.

Solution

[asy] size(9cm); pair A = dir(130);  pair B = dir(220);  pair C = dir(320);  draw(unitcircle, lightblue);  pair P = dir(-90);  pair Q = dir(90); pair D = extension(A, P, B, C);  pair O = origin;  pair M = extension(B, C, O, P);  pair N = 2*M-P;  draw(A--B--C--cycle, lightblue);  draw(A--P--Q, lightblue);  draw(A--N--D--O--A, lightblue);  draw(A--D--N--O--cycle, red);  dot("$A$", A, dir(A));  dot("$B$", B, dir(B));  dot("$C$", C, dir(C));  dot("$P$", P, dir(P));  dot("$Q$", Q, dir(Q));  dot("$D$", D, dir(225));  dot("$O$", O, dir(315));  dot("$M$", M, dir(315));  dot("$N$", N, dir(315)); [/asy]

Suppose ray $OM$ intersects the circumcircle of $BHC$ at $N'$, and let the foot of the A-altitude of $ABC$ be $E$. Note that $\angle BHE=90-\angle HBE=90-90+\angle C=\angle C$. Likewise, $\angle CHE=\angle B$. So, $\angle BHC=\angle BHE+\angle CHE=\angle B+\angle C$. $BHCN'$ is cyclic, so $\angle BN'C=180-\angle BHC=180-\angle B-\angle C=\angle A$. Also, $\angle BAC=\angle A$. These two angles are on different circles and have the same measure, but they point to the same line $BC$! Hence, the two circles must be congruent. (This is also a well-known result)

We know, since $M$ is the midpoint of $BC$, that $OM$ is perpendicular to $BC$. $AH$ is also perpendicular to $BC$, so the two lines are parallel. $AN$ is a transversal, so $\angle HAN=\angle ANO$. We wish to prove that $\angle ANO=\angle ADO$, which is equivalent to $AOND$ being cyclic.

Now, assume that ray $OM$ intersects the circumcircle of $ABC$ at a point $P$. Point $P$ must be the midpoint of $\stackrel{\frown}{BC}$. Also, since $AD$ is an angle bisector, it must also hit the circle at the point $P$. The two circles are congruent, which implies $MN=MP\implies ND=DP\implies$ NDP is isosceles. Angle ADN is an exterior angle, so $\angle ADN=\angle DNP+\angle DPO=2\angle DPO$. Assume WLOG that $\angle B>\angle C$. So, $\angle DPO=\angle APO=\frac{\angle B+\angle C}{2}-\angle C=\frac{\angle B-\angle C}{2}$. In addition, $\angle AON=\angle AOP=\angle AOB+\angle BOP=2\angle C+\angle A$. Combining these two equations, $\angle AON+\angle ADN=\angle B-\angle C+2\angle C+\angle A=\angle A+\angle B+\angle C=180$.

Opposite angles sum to $180$, so quadrilateral $AOND$ is cyclic, and the condition is proved.

-william122

See also

2017 USAJMO (ProblemsResources)
Preceded by
Problem 4
Followed by
Problem 6
1 2 3 4 5 6
All USAJMO Problems and Solutions